Added topic tags to some S&J-4 problems + minor typos
[course.git] / latex / problems / Serway_and_Jewett_4 / problem23.13.tex
1 \begin{problem*}{23.13} % rail guns
2 Figure P23.12 shows a top view of a bar that can slide without
3 friction.  The resistor is $R = 6.00\Omega$, and a $B = 2.50\U{T}$
4 magnetic field is directed perpendicularly downward, into the paper.
5 Let $l = 1.20\U{m}$.
6 \Part{a} Calculate the applied force required to move the bar to the
7 right at a constant speed $v = 2.00\U{m/s}$.
8 \Part{b} At what rate is energy delivered to the resistor?
9 \end{problem*}
10
11 \begin{solution}
12 \Part{a}
13 Let $x$ be the width of the enclosed loop.  The magnetic flux is then
14 \begin{equation}
15  \Phi_B = AB = xlB
16 \end{equation}
17 So the induced emf is
18 \begin{equation}
19  \varepsilon = -\frac{d\Phi_B}{dt} = -lB \frac{dx}{dt} = -lvB
20 \end{equation}
21 So the induced current is
22 \begin{equation}
23  I = \frac{\varepsilon}{R} = \frac{-lvB}{R}
24 \end{equation}
25 The $-$ sign indicates the current is counterclockwise (out of the
26 page), so current flows upward through the bar, so the magnetic force
27 on the bar is to the left, so our applied force must be \ans{to the right}.
28
29 The work begin done by the applied force is 
30 \begin{equation}
31  W = F \cdot dx \;.
32 \end{equation}
33 So the power input from the force is
34 \begin{equation}
35  P_F = \frac{W}{dt} = F \frac{dx}{dt} = Fv \;.
36 \end{equation}
37
38 All of this power must be dissipated by the resistor, so the current is
39 \begin{align}
40  P &= I^2 R \\
41  I &= \sqrt{\frac{P}{R}} = \sqrt{\frac{Fv}{R}} \;.
42 \end{align}
43
44 We combine both current equations to yield
45 \begin{align}
46  \frac{-lvB}{R} &= \sqrt{\frac{Fv}{R}} \\
47  (lvB)^2 &= RFv \\
48  F &= \frac{v(lB)^2}{R} = \ans{3.00\U{N}} \;.
49 \end{align}
50
51 \Part{b}
52 Going back and plugging in $F$,
53 \begin{equation}
54  P_F = Fv = \ans{6.00\U{W}} \;.
55 \end{equation}
56 \end{solution}